1answer.
Ask question
Login Signup
Ask question
All categories
  • English
  • Mathematics
  • Social Studies
  • Business
  • History
  • Health
  • Geography
  • Biology
  • Physics
  • Chemistry
  • Computers and Technology
  • Arts
  • World Languages
  • Spanish
  • French
  • German
  • Advanced Placement (AP)
  • SAT
  • Medicine
  • Law
  • Engineering
Fittoniya [83]
3 years ago
14

A power drill uses 13.73 amps of current and has a power rating of 1140 watts. What voltage does the drill require

Physics
2 answers:
Arlecino [84]3 years ago
7 0
I have the formula to find the voltage through the used of current and the power. 
Voltage = power / current        Voltage is measured in volts, the power is measured in watts and current is measured amps.

V = W/I

Given          W = 1140 watts  I = 13.73 amps

V = 1140 / 13.73
   = 83.03 Volts

My answer is that the drill required to have 83.03 volts. I think the choices above aren't reliable for this problem. 
diamong [38]3 years ago
5 0
7amps cause of how fast the drill is going
You might be interested in
Acceleration question plz help
andrezito [222]
For #5 It's helpful to draw a free body diagram so you know which way the forces are acting on the block.

the weight mg is acting downwards, and you need to find the vertical and horizontal components of mg using sin and cosine. so do 15x9.8xsin40 which is the force. Assuming no friction, this is the only force acting on the block, as the forces on the vertical plane cancel out i.e the normal force and weight of the block.

after, just do F=ma And since you know F and m, solve for a.
3 0
3 years ago
ou are out stargazing with your 13.4-cm telescope. You point your telescope at an interesting formation in the sky, which you th
Alinara [238K]

Answer:

θ = 4.716 10⁻⁶ rad

Explanation:

In order for the releases to be considered separate, they must meet the Rayleigh criterion that establishes that the maximum diffraction of one star must coincide with the first minimum of the diffraction pattern of the second star.

We use the diffraction equation for a slit

            a sin θ = m λ

The minimum occurs at m = 1

             sin θ = λ / a

Since the angles in these systems are very small, we can approximate the sine to its angle in radians

             θ = λ / a

The telescope has a circular aperture whereby polar cords should be used, which introduces a constant number

           θ = 1.22 λ / a

Let's calculate

          θ = 1.22 518 10⁻⁹ / 13.4 10⁻²

          θ = 4.716 10⁻⁶ rad

8 0
2 years ago
According to Coulomb's law, the electrical force between two charged objects:A.is zero if they are opposite charges.B.increases
kondor19780726 [428]

Answer and explanation:

A correct option is an option (B).

The electrical force between two charges is given as,

F=\frac{1}{4\pi\epsilon_0}\frac{q_1q_1}{r^2}

The electrical force is directly proportional to the product of two charges. Thus Force will depend on two charges irrespective of their signs.

Option (A) is incorrect because if charges are opposite, the value of force will not be zero. It will be -ve.

Option (C) is incorrect because the force is directly proportional to the product of charges, it depends on the amount of charge.

Option (D) is also incorrect because the force in inversly proportional to the distance between two charges. Thus, if the distance between two charges is increased, the force between two charges will decrease.

Concllusion:

The correct option is option (B).

7 0
9 months ago
An object has a mass of 50.0 g and a volume of 10.5 cm3. What is the object's density?
vagabundo [1.1K]
The density is 4.76 gcm^-3
and if mass is in kg then density is equal to 4.76*10^-3
3 0
2 years ago
Read 2 more answers
Nativism was rejected in the late 1600’s and early 1700’s by a group, of philosophers called:
Gelneren [198K]

Answer:

Nativism was rejected in the late 1600’s and early 1700’s by a group, of philosophers called?

Empiricists Is the Correct Answer!

xXxAnimexXx

Happy Labor day!

4 0
2 years ago
Other questions:
  • What is the relationship between the mass, velocity, and momentum of an object?
    8·2 answers
  • A car increases its speed from 9.6 meters per second to 11.2 meters per second in 4.0 seconds. The average acceleration of the c
    14·2 answers
  • You stand on a frictional platform that is rotating at 1.8 rev/s. Your arms are outstretched, and you hold a heavy weight in eac
    11·1 answer
  • A spring gun is made by compressing a spring in a tube and then latching the spring at
    7·1 answer
  • The order of elements in the periodic table is based on the atomic number.<br> true or false
    7·1 answer
  • What would happen if the earth’s mantle was completely solid? Why do you think so?
    15·1 answer
  • During a baseball game, a batter hits a high
    12·2 answers
  • Which statement correctly describes the movement of thermal energy according to the second law of thermodynamics? The natural te
    8·2 answers
  • What is the power dissipated by a 50.0 ohm resistor?
    11·1 answer
  • A satellite of mass M = 270kg is in circular orbit around the Earth at an altitude equal to the earth's mean radius (6370 km). A
    10·1 answer
Add answer
Login
Not registered? Fast signup
Signup
Login Signup
Ask question!